diff --git a/LinAlg2.tex b/LinAlg2.tex index e7ff51c..ed9a9b4 100644 --- a/LinAlg2.tex +++ b/LinAlg2.tex @@ -157,10 +157,10 @@ postaction={decorate}}, \section{Permutationen} \begin{defin} - Sei $n \in \mathbb{N} \setminus \{0\}, [n] := \{1, 2, \dots, n\}$. \\ + Sei $n \in \mathbb{N} \setminus \{0\}, [n] \coloneq \{1, 2, \dots, n\}$. \\ Eine bijektive Abbildung $\pi\colon[n]\to[n]$ heißt \underline{Permutation} von $[n]$. Wir definieren die \underline{symmetrische Gruppe} - $S_n := \{\pi\text{ Permutation von }[n]\}$ + $S_n \coloneq \{\pi\text{ Permutation von }[n]\}$ mit der Hintereinanderausführung als Gruppenoperation. \end{defin} @@ -231,7 +231,7 @@ postaction={decorate}}, (3 1 2) = (2 1)(3 1) = (3 1)(3 2) \] \item $f\in \mathbb{Z}[X_1, \dots, X_n], \pi \in S_n$ \\ - $\pi f(X_1, \dots, X_n) := f(X_{\pi(1)}, \dots, X_{\pi(n)})$ + $\pi f(X_1, \dots, X_n) \coloneq f(X_{\pi(1)}, \dots, X_{\pi(n)})$ \end{itemize} \subsubsection{Beispiel} $\pi = (2 3 1), f(X_1, X_2, X_3) = X_1-X_2+X_1X_3 \implies \pi f(X_1, X_2, X_3) = X_2 - X_3 + X_2X_1$ @@ -255,8 +255,8 @@ $\pi = (2 3 1), f(X_1, X_2, X_3) = X_1-X_2+X_1X_3 \implies \pi f(X_1, X_2, X_3) \begin{align*} \pi f(X_1, \dots, X_n) & = \prod_{i\pi(j)\}}} @@ -409,7 +409,7 @@ $\varphi$ alternierend und $a_i = a_j$ für $i\neq j \implies \varphi(a_1, \dots \lambda_{1j_1}\cdots\lambda_{nj_n}} \\ & \underbrace{=}_{\mathclap{\varphi\text{ alternierend}}} \sum_{\substack{j_1, \dots, j_n \\ - \text{paarweise verschieden}}} + \text{paarweise verschieden}}} {\varphi(b_{j_1}, \dots, b_{j_n})\lambda_{1j_1} \cdots \lambda_{nj_n}} \\ & = \sum_{\pi\in S_n} \varphi(b_{\pi(1)}, \dots, b_{\pi(n)}) \lambda_{1\pi(1)} \cdots \lambda_{n\pi(n)} \\ @@ -482,7 +482,7 @@ alternierende n-Linearform. \begin{proof} Sei $a_1, \dots, a_n$ Basis von V. Nach Lemma \ref{theo:1.2.4} ist $\varphi_i(a_1, \dots, a_n)\neq0, i=1, 2$.\\ - Sei $c:=\dfrac{\varphi_1(a_1, \dots, a_n)}{\varphi_2(a_1, \dots, a_n)} \in \K\setminus\{0\}$.\\ + Sei $c\coloneq\dfrac{\varphi_1(a_1, \dots, a_n)}{\varphi_2(a_1, \dots, a_n)} \in \K\setminus\{0\}$.\\ Sei $b_1, \dots, b_n$ mit $b_i=\sum\lambda_{ij}a_j$.\\ Dann gilt nach Satz \ref{theo:1.2.5}(a), dass für $i=1, 2$ \begin{align*} @@ -499,8 +499,8 @@ alternierende n-Linearform. Sei $B=(a_1, \dots, a_n)$ Basis des \K-Vektorraums V. Sei $\varphi$ nicht ausgeartete n-Linearform und $\alpha \in \homkv$. Dann ist die \underline{Determinante von $\alpha$} definiert durch \[ - \det(\alpha):=\det{}_\K(\alpha) - :=\frac{\varphi(\alpha(a_1), \dots, \alpha(a_n))}{\varphi(a_1, \dots, a_n)} + \det(\alpha)\coloneq\det{}_\K(\alpha) + \coloneq\frac{\varphi(\alpha(a_1), \dots, \alpha(a_n))}{\varphi(a_1, \dots, a_n)} \] \end{defin} @@ -519,7 +519,7 @@ alternierende n-Linearform. ausgeartet, \[\varphi(\alpha(a_1), \dots, \alpha(a_n))\neq0\] - Sei $\varphi_\alpha(b_1, \dots, b_n) := \varphi(\alpha(b_1), \dots, + Sei $\varphi_\alpha(b_1, \dots, b_n) \coloneq \varphi(\alpha(b_1), \dots, \alpha(b_n))$. Dann ist $\varphi_\alpha$ alternierend und nicht ausgeartet. Wegen Satz \ref{theo:1.2.6} folgt, dass $c\in\K\setminus\{0\}$ existiert mit \begin{equation} @@ -530,9 +530,10 @@ alternierende n-Linearform. Da \ref{eq:constantphi} unabhängig von B ist also $\det(\alpha)$ unabhängig von B. Sei nun $\psi$ eine zweite alternierende, nicht ausgeartete n-Form und - $\psi_\alpha(b_1, \dots, b_n) := \psi(\alpha(b_1), \dots, \alpha(b_n))$. Dann - ist $\psi_\alpha$ alternierend und nicht ausgeartet. Nach Satz \ref{theo:1.2.6} - gibt es $d\in\K\setminus\{0\} \text{ mit }d=\frac\psi\varphi$. Also gilt: + $\psi_\alpha(b_1, \dots, b_n) \coloneq \psi(\alpha(b_1), \dots, \alpha(b_n))$. + Dann ist $\psi_\alpha$ alternierend und nicht ausgeartet. Nach Satz + \ref{theo:1.2.6} gibt es $d\in\K\setminus\{0\} \text{ mit }d=\frac\psi\varphi$. + Also gilt: \[ \det(\alpha)=\frac{\varphi_\alpha(a_1, \dots, a_n)}{\varphi(a_1, \dots, a_n)}= \frac{d\varphi_\alpha(a_1, \dots, a_n)}{d\varphi(a_1, \dots, a_n)}= @@ -651,7 +652,7 @@ Schreibweise für $A=(a_{ij})$: \det(A^T) & = \sum_{\pi\in S_n}\sgn(\pi)a_{\pi(1)1}\cdots a_{\pi(n)n} \\ & =\sum_{\pi\in S_n}\sgn(\pi)a_{1\pi^{-1}(1)}\cdots a_{n\pi^{-1}(n)} \\ & \underbrace{=}_{\substack{\sgn(\pi^{-1})=\sgn(\pi) \\ - \pi^{-1}\mapsto\pi}} \sum_{\pi\in S_n} \sgn(\pi)a_{1\pi(1)}\cdots a_{n\pi(n)} + \pi^{-1}\mapsto\pi}} \sum_{\pi\in S_n} \sgn(\pi)a_{1\pi(1)}\cdots a_{n\pi(n)} \end{aligned} \end{equation*} \item[b) - i)] folgt daraus, dass für \[\alpha\colon @@ -741,7 +742,7 @@ $n>3 \to $ Gaußalgorithmus \begin{defin} Sei $A\in\K^{n\times n}$ und $i, j\in[n]$. Sei $M_{ij}\in\K^{n\times n}$ die Matrix, welche durch Ersetzung der j-ten Spalte durch den i-ten Einheitsvektor $e_j$ entsteht.\\ - $A_{ij}:=\det(M_{ij})$ heißt \underline{Kofaktor} (zum Indexpaar $(i, j)$). + $A_{ij}\coloneq\det(M_{ij})$ heißt \underline{Kofaktor} (zum Indexpaar $(i, j)$). \begin{equation*} \bordermatrix{ &&&&j \cr @@ -774,7 +775,7 @@ da obige Matrix aus $M_{ij}$ durch Spaltenadditionen hervorgeht. Sei $\tilde{A_{ij}}\in\K^{(n-1)\times(n-1)}$ die Matrix, welche aus A durch Streichung der i-ten Spalte und j-ten Zeile hervorgeht und - $D_{ij}:=\det(\tilde{A_{ij}})$. Dann gilt \[A_{ij}=(-1)^{i+j}D_{ij}\] + $D_{ij}\coloneq\det(\tilde{A_{ij}})$. Dann gilt \[A_{ij}=(-1)^{i+j}D_{ij}\] \end{lemma} \begin{proof} Transformiere durch ($i-1$) Spaltenvertauschungen und ($j-1$) Zeilenvertauschungen die Matrix @@ -916,7 +917,7 @@ da obige Matrix aus $M_{ij}$ durch Spaltenadditionen hervorgeht. $D\in \K^{n\times n}$ heißt \underline{Diagonalmatrix} wenn $\forall i\neq j\colon d_{ij}=0$. Wir schreiben auch \[ - \diag(\lambda_1, \dots, \lambda_n):= + \diag(\lambda_1, \dots, \lambda_n)\coloneq \begin{pmatrix} \lambda_1 & 0 & \dots & 0 \\ 0 & \lambda_2 & \dots & 0 \\ @@ -1016,10 +1017,10 @@ da obige Matrix aus $M_{ij}$ durch Spaltenadditionen hervorgeht. \begin{defin} \begin{enumerate}[label=\alph*)] \item Sei $\alpha \in \homkv$ und $\lambda \in \spec(\alpha)$. Dann heißt - $\eig_\alpha(\lambda):=\{v\in V\colon \alpha(v) = \lambda v \}$ der zugehörige - \underline{Eigenraum}. + $\eig_\alpha(\lambda)\coloneq\{v\in V\colon \alpha(v) = \lambda v \}$ der + zugehörige \underline{Eigenraum}. \item Sei $A \in \K^{n\times n}$ und $\lambda \in \spec(A)$. Dann heißt - $\eig_A(\lambda):=\{v\in \K^n\colon A v = \lambda v \}$ der zugehörige + $\eig_A(\lambda)\coloneq\{v\in \K^n\colon A v = \lambda v \}$ der zugehörige \underline{Eigenraum}. \end{enumerate} \end{defin} @@ -1167,15 +1168,15 @@ $\le\genfrac{}{}{0pt}{0}{\dim(V)}{n}$, da \right)} \\ & \left( \begin{array}{c c | c} - \bar1 & \bar4 & \bar0 \\ - \bar1 & \bar4 & \bar0 - \end{array} + \bar1 & \bar4 & \bar0 \\ + \bar1 & \bar4 & \bar0 + \end{array} \right) \\ & \left( \begin{array}{c c | c} - \bar1 & \bar4 & \bar0 \\ - \bar0 & \bar0 & \bar0 - \end{array} + \bar1 & \bar4 & \bar0 \\ + \bar0 & \bar0 & \bar0 + \end{array} \right) \\ & \implies \eig_{\bar2}(A) = \linspan{ \begin{pmatrix} @@ -1276,7 +1277,7 @@ $\le\genfrac{}{}{0pt}{0}{\dim(V)}{n}$, da \end{satz} \begin{proof} Klarerweise gilt $1\le m_g(\mu)$ da $\mu$ Eigenwert ist. - Sei $r:= m_g(\mu)$ und $b_1, \dots, b_r$ Basis von $\eig_\alpha(\mu)$. Sei $B=(b_1, \dots, b_n)$ Basis. + Sei $r\coloneq m_g(\mu)$ und $b_1, \dots, b_r$ Basis von $\eig_\alpha(\mu)$. Sei $B=(b_1, \dots, b_n)$ Basis. Dann ist \[ {}_B M(\alpha)_B = \bordermatrix{ @@ -1350,7 +1351,7 @@ $\le\genfrac{}{}{0pt}{0}{\dim(V)}{n}$, da \item[$\impliedby$:] Aus i), ii) folgt, dass \begin{equation} \label{eq:2.2.15.1} - \sum_{i=1}^r \underbrace{\dim(\eig_\alpha(\lambda_i))}_{=m_g(\lambda_i)=:d_i} = n + \sum_{i=1}^r \underbrace{\dim(\eig_\alpha(\lambda_i))}_{=m_g(\lambda_i)\eqcolon d_i} = n \end{equation} Sei $(b_i^1, \dots, b_i^{d_i})$ Basis von $\eig_\alpha(\lambda_i)$. Wir zeigen, dass $B=\{b_i^1, \dots, b_i^{d_i}\colon i=1, \dots, r\}$ Basis ist. @@ -1417,7 +1418,7 @@ $ \end{vmatrix} \\ \underbrace{=}_{\mathclap{\substack{\text{Entwicklung} \\ - \text{nach 1. Zeile}}}} + \text{nach 1. Zeile}}}} & (1-\lambda) \begin{vmatrix} -2 -\lambda & 1 \\ @@ -1444,31 +1445,31 @@ $ \begin{array}{c c c | c} 1-3 & 2 & 2 & 0 \\ 2 & -2-3 & 1 & 0 \\ 2 & 1 & -2-3 & 0 - \end{array} + \end{array} \right) = \left( \begin{array}{c c c | c} -2 & 2 & 2 & 0 \\ 2 & -5 & 1 & 0 \\ 2 & 1 & -5 & 0 - \end{array} + \end{array} \right) \\ & \sim \left( \begin{array}{c c c | c} -1 & 1 & 1 & 0 \\ 0 & -3 & 3 & 0 \\ 0 & 3 & -3 & 0 - \end{array} + \end{array} \right) \sim \left( \begin{array}{c c c | c} 1 & -1 & -1 & 0 \\ 0 & 1 & -1 & 0 \\ 0 & 0 & 0 & 0 - \end{array} + \end{array} \right) \sim \left( \begin{array}{c c c | c} 1 & 0 & -2 & 0 \\ 0 & 1 & -1 & 0 \\ 0 & 0 & 0 & 0 - \end{array} + \end{array} \right) \\ & \implies \eig_A(3) = \linspan{ \begin{pmatrix} @@ -1485,19 +1486,19 @@ $ \begin{array}{c c c | c} 1+3 & 2 & 2 & 0 \\ 2 & -2+3 & 1 & 0 \\ 2 & 1 & -2+3 & 0 - \end{array} + \end{array} \right) = \left( \begin{array}{c c c | c} 4 & 2 & 2 & 0 \\ 2 & 1 & 1 & 0 \\ 2 & 1 & 1 & 0 - \end{array} + \end{array} \right) \\ & \sim \left( \begin{array}{c c c | c} 2 & 1 & 1 & 0 \\ 0 & 0 & 0 & 0 \\ 0 & 0 & 0 & 0 - \end{array} + \end{array} \right) \\ & \implies \eig_A(-3) = \linspan { \begin{pmatrix} @@ -1536,7 +1537,7 @@ $ \begin{lemma} \label{theo:2.2.16} Sei $A\in\K^{\nxn}$ und $\underbrace{\spur(A)}_{\mathclap{\color{red}\text{\dq Spur von $A$ \dq}}} - := \sum\limits_{i=1}^n a_{ii}$ + \coloneq \sum\limits_{i=1}^n a_{ii}$ \[\chi_A(\lambda) = (-1)^n\lambda^n + (-1)^{n-1} \spur(A) \lambda^{n-1} + \cdots + \det(A)\] \end{lemma} \begin{proof} @@ -1576,7 +1577,7 @@ $ $\chi_A(\lambda)=c_n \lambda^n + c_{n-1} \lambda^{n-1} + \cdots + c_0$. Dann gilt \[ - \chi_A(A):=c_n A^n + c_{n-1} A ^{n-1} + \cdots c_0 I = 0 = + \chi_A(A)\coloneq c_n A^n + c_{n-1} A ^{n-1} + \cdots c_0 I = 0 = \begin{pmatrix} 0 & \dots & 0 \\ \vdots & \ddots & \vdots \\ @@ -1586,7 +1587,7 @@ $ \] \end{satz} \begin{proof} - Sei $B := A^T - \lambda I = + Sei $B \coloneq A^T - \lambda I = \begin{pmatrix} a_{11} - \lambda & a_{21} & \dots & a_{n1} \\ a_{12} & a_{22} - \lambda & \dots & a_{n2} \\ @@ -1594,7 +1595,7 @@ $ a_{1n} & a_{2n} & \dots & a_{nn} - \lambda \end{pmatrix} = (a_{ji} - \delta_{ij} \lambda)_{ij}$ - und $C:= \adj(B)$, sodass + und $C\coloneq \adj(B)$, sodass \begin{equation} CB \overset{\text{\ref{theo:1.4.7}}}{=} \det(B) I_n = \chi_A \cdot I_n \; [\chi_A = \chi_{A^T}] \label{eq:2.2.18.1} @@ -1628,15 +1629,15 @@ Sei $f(\lambda) \underbrace{=}_{\text{(*)}} \prod\limits_{j=1}^{n}(\lambda_j - \lambda) = \underbrace{c_n}_{=(-1)^n}\lambda^n + c_{n-1}\lambda ^{n-1} + \cdots + c_0$ Wie können wir $c_j$ effizient bestimmen? \[ - \sigma_j := (-1)^j \sum\limits_{\substack{S\subset [n] \\ \abs{ S } = n-j}} + \sigma_j \coloneq (-1)^j \sum\limits_{\substack{S\subset [n] \\ \abs{ S } = n-j}} \prod\limits_{s \in S} \lambda_s \] \begin{itemize} \item [Bemerkung 1:] $\displaystyle { c_j = (-1)^{j} \sum_{\substack{S\subseteq [n] \\ - \abs{ S } = n-j}} \prod_{s \in S} \lambda_s =: + \abs{ S } = n-j}} \prod_{s \in S} \lambda_s \eqcolon \sigma_{n-j}^n (\lambda_1, \dots, \lambda_n)}$ \\ Dies folgt aus (*) durch Ausmultiplizieren \\ - Sei nun weiters $p_j^n(\lambda_1, \dots, \lambda_n) := \sum\limits_{i=1}^{n}\lambda_i^j$ + Sei nun weiters $p_j^n(\lambda_1, \dots, \lambda_n) \coloneq \sum\limits_{i=1}^{n}\lambda_i^j$ \item [Bemerkung 2:] $\sigma_j^n, p_j^n$ sind symmetrisch, das heißt \[ \begin{aligned} @@ -1666,7 +1667,7 @@ Sei $f(\lambda) \underbrace{=}_{\text{(*)}} \prod\limits_{j=1}^{n}(\lambda_j - folgt $\sigma_n^n p_0^n + \sum\limits_{j=0}^n \sigma_j^n p_{n-j}^n = 0$ was mit $p_0^n = n$ die gewünschte Aussage liefert. \item [$k] (0, 0) --node[above]{$a_1$} (\a1, \a2); \draw [->] (0, 0) --node[above]{$a_2$} (\a3, \a4); \draw [->, blue, thick] (0, 0) --node[below right]{$\inner{a_2}{b_1}b_1$} (\t1, \t2); - \draw [->, violet, very thick] (0, 0) --node[below right]{$\frac{a_1}{\norm{a_1}}=:b_1$} (\normeda1, \normeda2); - \draw [->, magenta, thick] (0, 0) --node[left]{$c_2:=a_2 - \inner{a_2}{b_1}b_1$} (\c1, \c2); - \draw [->, teal, very thick] (0, 0) --node[right]{$\frac{c_2}{\norm{c_2}}=:b_2$} (\b3, \b4); + \draw [->, violet, very thick] (0, 0) --node[below right]{$\frac{a_1}{\norm{a_1}}\eqcolon b_1$} (\normeda1, \normeda2); + \draw [->, magenta, thick] (0, 0) --node[left]{$c_2\coloneq a_2 - \inner{a_2}{b_1}b_1$} (\c1, \c2); + \draw [->, teal, very thick] (0, 0) --node[right]{$\frac{c_2}{\norm{c_2}}\eqcolon b_2$} (\b3, \b4); \end{tikzpicture} \subsubsection{Beispiel} @@ -2891,7 +2892,7 @@ $ $[M = {v} \implies v \bot N]$ \item Für $M \subseteq V$ heißt \[ - M^\bot := \{ v\in V\colon v \bot M \} = \{ v \in V\colon \forall w \in M\colon \inner vw = 0 \} + M^\bot \coloneq \{ v\in V\colon v \bot M \} = \{ v \in V\colon \forall w \in M\colon \inner vw = 0 \} \] \underline{orthogonales Komplement} von M \end{itemize} @@ -2948,17 +2949,17 @@ Wir zeigen: $U^\bot = \{0\} \implies (U^\bot)^{{}^\bot} =V\neq U$ Beweis wird hier nicht geführt. \end{nonumbersatz} \par -Sei $f \in V \setminus \{0\}, a := \norm f^2 = \inner ff, b = \norm f _\infty$. -Sei $p \in U\colon \norm{f-p}_\infty < \frac a {2b}$ \\ Behauptung: $\inner fp - > 0$ +Sei $f \in V \setminus \{0\}, a \coloneq \norm f^2 = \inner ff, b = \norm f + _\infty$. Sei $p \in U\colon \norm{f-p}_\infty < \frac a {2b}$ \\ Behauptung: +$\inner fp > 0$ \begin{align*} \inner fp = \int_0^1 f(t)p(t) dt & = \int_0^1 f(t)[f(t) - (f(t) - p(t))]dt \\ & = \int_0^1 f(t)f(t)dt - \int_0^1 f(t)(f(t) - p(t)) dt \\ & = a - \int_0^1 f(t)[f(t)-p(t)] dt \\ \int_0^1 \underbrace{f(t)}_{\substack{\le \norm b_\infty \\ - \le b}} + \le b}} [\underbrace{f(t) - p(t)}_{\substack{\le \norm{f - p}_\infty \\ - \le \frac a {2b}}}] + \le \frac a {2b}}}] & \le \int_0^1 b \cdot \frac a {2b}dt - \int_0^1 \frac a2 dt = \frac a2 \\ \forall f \in V\colon \exists p \in U\colon \int_0^1 f(t)p(t)dt @@ -2984,7 +2985,7 @@ Sei $p \in U\colon \norm{f-p}_\infty < \frac a {2b}$ \\ Behauptung: $\inner fp Beispiel: $U, V$ wie vorher. $\alpha \in \Hom_\R(U, V), \alpha(p) = p \forall p \in U$ \\ Angenommen $\exists \alpha^* \in \Hom_\R(V, U), e(t) = e^t \implies e \in V$ \\ $\alpha^* (e) = a_1 p_1 + \dots + a_m p_m$ \\ - $f := e - (a_1 p_1 + \dots + a_m p_m) = e- \alpha^*(e) \neq 0$ \\ + $f \coloneq e - (a_1 p_1 + \dots + a_m p_m) = e- \alpha^*(e) \neq 0$ \\ Behauptung: $f \in U^\bot (\implies f = 0$ \Lightning) \item $i \in \{m+1, m+2, \dots \}$ %Eigentlich nicht so wirklich ein Punkt?! \begin{align*} @@ -3005,7 +3006,7 @@ Sei $p \in U\colon \norm{f-p}_\infty < \frac a {2b}$ \\ Behauptung: $\inner fp Mit $\{e_1, \dots, e_n\}$ Orthonormalbasis von $\underbrace{V}_{ \mathclap{\text{Existenz gegeben wegen Satz \ref{theo:3.1.17}}}}$ gilt: \[ - \alpha^*(w) := \sum_{i=1}^n \inner{w}{\alpha(e_i)} e_i + \alpha^*(w) \coloneq \sum_{i=1}^n \inner{w}{\alpha(e_i)} e_i \] \end{lemma} \begin{proof} @@ -3022,8 +3023,8 @@ Sei $p \in U\colon \norm{f-p}_\infty < \frac a {2b}$ \\ Behauptung: $\inner fp \begin{defin} Sei $A \in \C^{m \times n}$. \begin{align*} - & \overline{A} := (\overline{a}_{ij})_{i,j} & & \text{ zu $A$ \underline{konjugiert komplexe} Matrix} \\ - & A^* = (\overline{A})^T & & \text{ zu $A$ \underline{adjungierte} Matrix} + & \overline{A} \coloneq (\overline{a}_{ij})_{i,j} & & \text{ zu $A$ \underline{konjugiert komplexe} Matrix} \\ + & A^* = (\overline{A})^T & & \text{ zu $A$ \underline{adjungierte} Matrix} \end{align*} \end{defin} @@ -3042,7 +3043,7 @@ Sei $p \in U\colon \norm{f-p}_\infty < \frac a {2b}$ \\ Behauptung: $\inner fp \begin{proof} \begin{align*} & A = {}_F M(\alpha)_E = (a_{ij})_{\substack{i=1,\dots,m \\ - j=1,\dots,n}}, \, + j=1,\dots,n}}, \, B = {}_E M(\alpha^*)_F = (b_{ij})_{\substack{i=1,\dots,n \\ j=1,\dots,m}} \\ & \alpha(e_j) = \sum_{i=1}^m a_{ij} f_i \\ @@ -3067,7 +3068,7 @@ Sei $p \in U\colon \norm{f-p}_\infty < \frac a {2b}$ \\ Behauptung: $\inner fp \item $\inner{\alpha(v)}{w} = \inner{v}{\alpha^*(w)} = \overline{\inner{\alpha^*(w)}{v}} =$ \\ $\overline{\inner{w}{(\alpha^*)^{{}^*}(v)}} = \inner{(\alpha^*)^{{}^*}(v)}{w} \; \forall v \in V, w \in W$ \\ $\implies \inner{\alpha(v) - (\alpha^*)^{{}^*}(v)}{w} = 0 \; \forall v \in V, w \in W, \; - w:= \alpha(v) - (\alpha^*)^{{}^*}(v)$ \\ + w\coloneq \alpha(v) - (\alpha^*)^{{}^*}(v)$ \\ $\implies \inner{\alpha(v) - (\alpha^*)^{{}^*}(v)}{\alpha(v) - (\alpha^*)^{{}^*}(v)} = 0 \iff \norm{\alpha(v) - (\alpha^*)^{{}^*}(v)} = 0 \implies \forall v \in V\colon \alpha(v) = (\alpha^*)^{{}^*}(v)$ \item @@ -3191,7 +3192,7 @@ Sei $p \in U\colon \norm{f-p}_\infty < \frac a {2b}$ \\ Behauptung: $\inner fp \end{align*} \begin{align*} & \overset{\mathclap{\substack{\text{Induktionsvorraussetzung} \\ - |}}} + |}}} {\implies} \exists \text{ ONB } (e_2, \dots, e_n) \text{ von $U$ aus Eigenvektoren von } \alpha \\ & \implies (e_1, \dots, e_n) \text{ ist ONB von $V$ aus Eigenvektoren von } \alpha @@ -3540,7 +3541,7 @@ Das sind genau die Längen- und Winkelerhaltenden Abbildungen. \item $v = 0 \implies \inner{\alpha(v)}{\alpha(w)} = \inner{0}{\alpha(w)} = 0 \checkmark$ \item $w = \lambda v \implies \inner{\alpha(w)}{\alpha(v)} = \lambda \inner{\alpha(v)}{\alpha(v)} = \lambda \norm{\alpha(v)}^2$. \\ - Sei $l := \frac{v}{\norm v} \overset{\text{d)}}{\implies} \alpha(l)$ ist ONS + Sei $l \coloneq \frac{v}{\norm v} \overset{\text{d)}}{\implies} \alpha(l)$ ist ONS $\implies \norm{\alpha(l)} = 1 \implies \norm{\alpha(v)} = \norm v$. \\ Es folgt $\inner{\alpha(v)}{\alpha(w)} = \inner vw \checkmark$. \item $v, w$ linear unabhängig. Sei $(e_1, e_2)$ ONS mit $\linspan{\{e_1, e_2\}} @@ -3601,7 +3602,7 @@ Das sind genau die Längen- und Winkelerhaltenden Abbildungen. \item Folgt direkt aus Satz~\ref{theo:3.3.2}: \begin{align*} \underset{\substack{\rotatebox{90}{=} \\ - u+iv}}{\norm{v_\C}} + u+iv}}{\norm{v_\C}} = 1 & \iff \norm u^2 + \norm v^2 = 1 \\ & \implies \norm{\alpha_\C(v_\C)} = \norm{\alpha(u)}^2 + \norm{\alpha(v)}^2 = 1 \end{align*} @@ -3613,8 +3614,8 @@ Das sind genau die Längen- und Winkelerhaltenden Abbildungen. \begin{itemize} \item $A \in \R^{\nxn}$ heißt \underline{orthogonal} wenn $A^{-1} = A^T$. \item $A \in \C^{\nxn}$ heißt \underline{unitär} wenn $A^{-1} = A^* = \overline{A}^T$. - \item $O(n, \R) := \{ A \in \R^{\nxn}\colon \det(A)\neq 0 \land A^{-1} = A^T \}$ - \item $U(n, \C) := \{ A \in \C^{\nxn}\colon \det(A)\neq 0 \land A^{-1} = A^* \}$ + \item $O(n, \R) \coloneq \{ A \in \R^{\nxn}\colon \det(A)\neq 0 \land A^{-1} = A^T \}$ + \item $U(n, \C) \coloneq \{ A \in \C^{\nxn}\colon \det(A)\neq 0 \land A^{-1} = A^* \}$ \end{itemize} \end{defin} @@ -3808,7 +3809,7 @@ Betrag 1 $\cong$ unitär, positiv $\cong$ selbstadjungiert mit positiven Eigenwe $\implies \gamma = \gamma' \implies \beta = \beta'$ \\ \underline{$\alpha$ nicht injektiv}: \begin{itemize} - \item $W := \ker(\alpha)^\bot \implies \alpha|_W$ ist injektiv. \\ + \item $W \coloneq \ker(\alpha)^\bot \implies \alpha|_W$ ist injektiv. \\ Sei $v, w \in W, \alpha(v) = \alpha(w) \implies \alpha(v - w) = 0 \implies v - w \in \ker(\alpha) = W^\bot \cap W = \{0\} \implies v = w$ $\implies \alpha|_W = \beta_W \circ \gamma_W$ mit $\beta, \gamma \in \Hom(W, W); \beta_W$ unitär, @@ -3829,13 +3830,13 @@ Betrag 1 $\cong$ unitär, positiv $\cong$ selbstadjungiert mit positiven Eigenwe & \inner{v}{\pi(w)} = \inner{\sum_{i=1}^n \lambda_i e_i}{\sum_{j=1}^k \mu_j e_j} = \sum_{i=1}^k \lambda_i \overline{\mu_i} \end{align*} - $\gamma := \pi^* \circ \gamma_W \circ \pi = \pi \circ \gamma_W \circ \pi$ + $\gamma \coloneq \pi^* \circ \gamma_W \circ \pi = \pi \circ \gamma_W \circ \pi$ \begin{align*} & v\in W & & \implies \gamma(v) = \gamma_W(v) \\ % Hier fehlt noch was aus der VO, nachschauen & v \in W^\bot = \ker(\alpha) & & \implies \gamma(v) = 0 \end{align*} - $\beta := \underset{W}{\beta_W} \oplus \underset{W^\bot}{I}$ ist orthogonal/unitär. \\ + $\beta \coloneq \underset{W}{\beta_W} \oplus \underset{W^\bot}{I}$ ist orthogonal/unitär. \\ $\implies \alpha = \beta \circ \gamma$ \end{itemize} \end{proof} @@ -3967,7 +3968,7 @@ Polarzerlegung $A \in \K^{\nxn}, A^* A$ symmetrisch/hermitesch Sei $V$ ein reeller/komplexer Vektorraum mit $\dim(V) = n, B=(b_1, \dots, b_n)$ Basis. Für $A \in K^{\nxn}$ ist \begin{equation} \label{eq:3.4.5.1} - \inner vw := {}_B \Phi(v)^T A {}_B \overline{\Phi(w)} + \inner vw \coloneq {}_B \Phi(v)^T A {}_B \overline{\Phi(w)} \end{equation} Genau dann ein Skalarprodukt, wenn $A$ symmetrisch/hermitesch und\\ $\underbrace{\text{positiv definit}}_{\substack{\forall \lambda \in \spec(A)\colon \lambda > 0 \\ @@ -3999,10 +4000,10 @@ Polarzerlegung $A \in \K^{\nxn}, A^* A$ symmetrisch/hermitesch \implies & a_{ij} = \overline{a_{ji}} \implies A \text{ hermitesch} \end{align*} Weiters muss $A$ positiv definit sein: Angenommen $\exists x \in \C^n \setminus \{0\}\colon x^T Ax = 0 - \implies v := \sum x_i b_i$, das heißt ${}_B \Phi(v) = x$ erfüllt $\inner vv = {}_B \Phi(v)^T A {}_B + \implies v \coloneq \sum x_i b_i$, das heißt ${}_B \Phi(v) = x$ erfüllt $\inner vv = {}_B \Phi(v)^T A {}_B \overline{\Phi(v)} = x^T A \overline x = 0$ \\ Sei $A$ hermitesch \& positiv definit. Klarerweise gilt dann für \\ - $\inner uv := {}_B \Phi(u)^T A {}_B \overline{\Phi(v)}$: + $\inner uv \coloneq {}_B \Phi(u)^T A {}_B \overline{\Phi(v)}$: \begin{align*} \inner{u+v}{w} & = \inner uw + \inner vw \\ \inner uv & = \overline{\inner vu} \\ @@ -4015,7 +4016,7 @@ Polarzerlegung $A \in \K^{\nxn}, A^* A$ symmetrisch/hermitesch \begin{align*} & \inner vv={}_B \Phi(v)^T A {}_B \overline{\Phi(v)} = {}_B \Phi(v)^T U^* \Sigma U {}_B \overline{\Phi(v)} = \underset{\substack{\rotatebox{90}{$=$} \\ - \sum \lambda_i \abs{ x_i }^2 > 0}} + \sum \lambda_i \abs{ x_i }^2 > 0}} {(\overline U {}_B\Phi(v))^T \Sigma \overline{\overline U {}_B (v)}} \\ & v \neq0 \implies \overline U {}_B \Phi(v) = \left( @@ -4030,7 +4031,7 @@ Polarzerlegung $A \in \K^{\nxn}, A^* A$ symmetrisch/hermitesch Sei $A \in \K^{\nxn}$ eine symmetrische/hermitesche Matrix. \begin{itemize} \item \[ - t(A) := \abs{ \{\lambda \in \spec(A)\colon \lambda > 0 \} } + t(A) \coloneq \abs{ \{\lambda \in \spec(A)\colon \lambda > 0 \} } \] heißt \underline{Trägheitsindex} von $A$. \item $A, B$ heißen \underline{kongruent} wenn eine invertierbare Matrix $Q \in \K^{\nxn}$ existiert mit @@ -4072,7 +4073,7 @@ $M_B(\sigma), M_{B'}(\sigma)$ sind kongruent (\& umgekehrt) & & & & & 1 \end{pmatrix} $ - $\implies S := PT$ ist invertierbar. + $\implies S \coloneq PT$ ist invertierbar. \begin{align*} S^* A S & = T \underbrace{P^* A P}_{\mathclap{\diag{\lambda_1, \dots, \lambda_n}}} T = T \begin{pmatrix} @@ -4113,17 +4114,17 @@ $M_B(\sigma), M_{B'}(\sigma)$ sind kongruent (\& umgekehrt) \end{pmatrix} = G \end{align*} - $t(A) := t, t(B) := s$ \\ + $t(A) \coloneq t, t(B) \coloneq s$ \\ Ordne so, dass $\ontop{\lambda_1, \dots, \lambda_t > 0, \lambda_{t+1}, \dots, \lambda_r < 0, \lambda_{r+1}, \dots, \lambda_n = 0} {\mu_1, \dots, \mu_s > 0, \mu_{s+1}, \dots, \mu_r < 0, \mu_{r+1}, \dots, \mu_n =0}$ \\ - Setze $a_i := \sqrt{\abs{\lambda_i}}, b_i := \sqrt{\abs{\mu_i}}$ + Setze $a_i \coloneq \sqrt{\abs{\lambda_i}}, b_i \coloneq \sqrt{\abs{\mu_i}}$ \begin{equation} \label{eq:3.4.6.1} x^* D x = \sum_{j=1}^t a_j^2 \abs{ x_j }^2 - \sum_{j=t+1}^r a_j^2 \abs{x_j}^2 \end{equation} - $C := P_2 Q^{-1} P_1, y := Cx$ + $C \coloneq P_2 Q^{-1} P_1, y \coloneq Cx$ \begin{equation} \label{eq:3.4.6.2} x^* D x = x^* C^* G C x = y^* G y = \sum_{j=1}^s b_j^2 \abs{ y_j }^2 - \sum_{j=s+1}^r b_j^2 @@ -4203,9 +4204,9 @@ Sei $A$ symmetrisch hermitesch, $\det(A) \neq 0 \implies \chi_A(\lambda) = a_1 \end{pmatrix} \end{equation} $V=\R^2$ - $ q(x) := \sigma(x, x) = x_1^2 + x_1 x_2 + x_2 x_1 - 5x_2^2, \R^2 \to \R$ + $ q(x) \coloneq \sigma(x, x) = x_1^2 + x_1 x_2 + x_2 x_1 - 5x_2^2, \R^2 \to \R$ \end{itemize} -Sei $ B= (b_1, \dots, b_n)$ Basis, so ist $M_B(\sigma) := (\sigma(b_i, b_j))_{i,j=1}^n$ +Sei $ B= (b_1, \dots, b_n)$ Basis, so ist $M_B(\sigma) \coloneq (\sigma(b_i, b_j))_{i,j=1}^n$ \begin{lemma} \label{theo:3.5.2} @@ -4365,7 +4366,7 @@ $ \implies q(\tilde x_1, \tilde x_2) = \lambda_1 \tilde x_1^2 + \lambda_2 \tilde Sei $\rho\colon V \to \K$ heißt \underline{quadratische Form} wenn $\forall u, v \in V, \lambda \in \K\colon$ \begin{enumerate}[label=\alph*)] \item $\rho(\lambda v) = \lambda^2 \rho(v)$ - \item $ \sigma(u, v) := \rho(u + v) - \rho(u) - \rho (v)$ ist eine (symmetrische) Bilinearform + \item $ \sigma(u, v) \coloneq \rho(u + v) - \rho(u) - \rho (v)$ ist eine (symmetrische) Bilinearform \end{enumerate} \end{defin} @@ -4378,7 +4379,7 @@ $ \implies q(\tilde x_1, \tilde x_2) = \lambda_1 \tilde x_1^2 + \lambda_2 \tilde $\rho$ quadratische Form $\implies \sigma(v, w) = \rho(u + v) - \rho(u) - \rho(v)$ ist symmetrische Bilinearform. \\ Sei umgekehrt $\sigma$ symmetrische Bilinearform, - $\rho(v) := \underset{\mathclap{\substack{\rotatebox{90}{$\to$}\\\operatorname{char}(\K) \neq 2}}} + $\rho(v) \coloneq \underset{\mathclap{\substack{\rotatebox{90}{$\to$}\\\operatorname{char}(\K) \neq 2}}} {\frac 12} \sigma(v, v)$. \begin{align*} \rho(\lambda v) = \frac 12 \sigma(\lambda v, \lambda v) & = \lambda^2 \frac 12 \sigma(v, v) = @@ -4401,7 +4402,7 @@ $ \implies q(\tilde x_1, \tilde x_2) = \lambda_1 \tilde x_1^2 + \lambda_2 \tilde \begin{enumerate}[label=\alph*)] \item $\rho(\lambda v) = \abs{\lambda}^2 \rho(v)$ \item $\rho(u+v) + \rho(u -v) = 2(\rho(u) + \rho(v))$ - \item $\sigma(u,v) := \frac 12 (\rho(u+v) + i\rho(u +iv) - (1+i)(\rho(u) + \rho(v)))$ ist hermitesche + \item $\sigma(u,v) \coloneq \frac 12 (\rho(u+v) + i\rho(u +iv) - (1+i)(\rho(u) + \rho(v)))$ ist hermitesche Sesquilinearform. \end{enumerate} \end{defin} @@ -4411,7 +4412,7 @@ $ \implies q(\tilde x_1, \tilde x_2) = \lambda_1 \tilde x_1^2 + \lambda_2 \tilde \end{lemma} \begin{proof} Für hermitesche Form ist durch Definition \ref{theo:3.5.6} c) eine hermitesche Sesquilinearform definiert. \\ -Sei umgekehrt $\sigma$ hermitesche Sesquilinearform. Dann ist $\rho(v) := \frac12 \sigma(v, v)$ hermitesche +Sei umgekehrt $\sigma$ hermitesche Sesquilinearform. Dann ist $\rho(v) \coloneq \frac12 \sigma(v, v)$ hermitesche Form: \begin{enumerate}[label=\alph*)] \item \checkmark @@ -4491,7 +4492,7 @@ zusammensetzen. \item $A^* A \in \K^{\nxn}$ selbstadjungiert und positiv semi-definit. \\ Eigenwerte $\lambda_1, \dots, \lambda_n \in [0, \infty)$, ONB $b_1, \dots, b_n$ aus Eigenvektoren. Sei $\lambda_1, \dots, \lambda_r \in (0, \infty), \lambda_{r+1} = \dots = \lambda_n = 0$ - $s_i := \sqrt{\lambda_i}, i\in [n]$ + $s_i \coloneq \sqrt{\lambda_i}, i\in [n]$ \item Es gilt, dass $\overbrace{\frac 1{s_1} A b_1}^{b_1'}, \dots, \overbrace{\frac 1{s_r} A b_r}^{b_r'}$ Orthonormalsystem in $\K^m$ ist. \begin{align*} @@ -4543,23 +4544,23 @@ $\implies \ker(\alpha) = \linspan{ b_{r+1}, \dots, b_n }_V, \im(\alpha) = \linsp \sum_{i=1}^r s_i \lambda_i b_i' \\ \left( \begin{smallmatrix} - x_1 \\ \vdots \\ x_n - \end{smallmatrix} + x_1 \\ \vdots \\ x_n + \end{smallmatrix} \right) & \mapsto \left( \begin{smallmatrix} - x_1 \\ \vdots \\ x_r - \end{smallmatrix} + x_1 \\ \vdots \\ x_r + \end{smallmatrix} \right) & & \mapsto \left( \begin{smallmatrix} - s_1 x_1 \\ \vdots \\ s_r x_r - \end{smallmatrix} + s_1 x_1 \\ \vdots \\ s_r x_r + \end{smallmatrix} \right) & & \mapsto \left.\left( \begin{smallmatrix} - s_1 x_1 \\ \vdots \\ s_r x_r \\ 0 \\ \vdots \\ 0 - \end{smallmatrix} + s_1 x_1 \\ \vdots \\ s_r x_r \\ 0 \\ \vdots \\ 0 + \end{smallmatrix} \right)\right\} m \\ \sum_{i=1}^r \frac{\mu_i}{s_i} b_i & \mapsfrom \sum_{i=1}^r \frac{1}{s_i} \mu_i b_i & & \underset{\beta^{-1}}{\mapsfrom} @@ -4658,8 +4659,8 @@ Wir haben eine echte Verallgemeinerung. $\im(\alpha) \supseteq \im(\alpha \circ \alpha^\dagger) \supseteq \im(\alpha \circ \alpha^\dagger \circ \alpha) = \im(\alpha) \implies \im(\alpha) = \im(\alpha \circ \alpha^\dagger)$ - \item $\nu := \alpha^\dagger \circ \alpha$ erfüllt $\nu \circ \nu$ und ist selbstadjungiert - für $\nu' := \alpha \circ \alpha^\dagger$ \\ + \item $\nu \coloneq \alpha^\dagger \circ \alpha$ erfüllt $\nu \circ \nu$ und ist selbstadjungiert + für $\nu' \coloneq \alpha \circ \alpha^\dagger$ \\ $\implies \underbrace{\ker(\nu)}_{=\ker(\alpha)} \bot \im(\nu)$ [Sei $v\in \ker(\nu), w = \nu(v) \in \im(\nu) \implies \inner vw = \inner{\nu(v)}{w} = 0$] \\ $\implies$ @@ -4798,7 +4799,7 @@ Wir haben eine echte Verallgemeinerung. \implies \alpha(w) \in \im(\alpha)^\bot \cap \im(\alpha) \implies \alpha(w) = 0 \\ \overset{\alpha \text{ injektiv}}{\implies} w = 0 & \text{\Lightning} \end{align*} - $\implies \beta:= (\alpha^* \circ \alpha)^{{}^{-1}} \circ \alpha^*$ ist wohldefiniert. + $\implies \beta\coloneq (\alpha^* \circ \alpha)^{{}^{-1}} \circ \alpha^*$ ist wohldefiniert. Nun gilt: \begin{itemize} \item $\alpha \circ \beta \circ \alpha = \alpha \circ (\alpha^* \circ \alpha)^{{}^{-1}} \circ \alpha^* \circ @@ -4912,7 +4913,7 @@ $\norm x = 1 \implies \norm{Ax} \ge s_1$ $\norm{b_i} \implies \lambda_1, \lambda_2 = \dots = \lambda_n = 0 \implies \norm{Ab_1} = s_1 \implies b_1$ löst unser Minimierungsproblem. \\ $Q = \{(x,y) \in \R^2\colon \psi(x, y) = 0\}$ \\ -$\psi(x, y):= a_1 x^2 + a_2 xy + a_3 y^2 a_4 x + a_5 y + a_6$ \\ +$\psi(x, y)\coloneq a_1 x^2 + a_2 xy + a_3 y^2 a_4 x + a_5 y + a_6$ \\ Gegeben: $(x_i,y_i)^m_{i=1}$ Suche $x = (a_1, \dots, a_6)^T$ mit $\norm x = 1$ sodass \[ \sum_{i=1}^m \left(a_1 x_i^2 + a_2 x_i y_i + a_3 y_i^2 + a_4 x_i + a_5 y_i + a_6\right)^2=\norm{Ax}^2